Fórum de Matemática
DÚVIDAS? Nós respondemos!

Um Fórum em Português dedicado à Matemática
Data/Hora: 03 jun 2024, 21:39

Os Horários são TMG [ DST ]




Fazer Nova Pergunta Responder a este Tópico  [ 4 mensagens ] 
Autor Mensagem
 Título da Pergunta: Provar que o limite não existe
MensagemEnviado: 23 mai 2013, 00:48 
Offline

Registado: 11 mar 2013, 20:36
Mensagens: 8
Localização: Rio de Janeiro
Agradeceu: 0 vez(es)
Foi agradecido: 0 vez(es)
lim sen(1/x²)
x->0


Topo
 Perfil  
 
MensagemEnviado: 23 mai 2013, 01:46 
Offline

Registado: 05 jan 2011, 12:35
Mensagens: 2235
Localização: Lisboa
Agradeceu: 683 vezes
Foi agradecido: 346 vezes
se fizer uma mudança de variável \(y=1/x\) fica com

\(\lim_{y \to +\infty} sen(y^2)\) que não tem limite

(não sei se quer a prova pela definição formal)

_________________
João Pimentel Ferreira
 
Partilhe dúvidas e resultados, ajude a comunidade com a sua pergunta!
Não lhe dês o peixe, ensina-o a pescar (provérbio chinês)
Fortalecemos a quem ajudamos pouco, mas prejudicamos se ajudarmos muito (pensamento budista)


Topo
 Perfil  
 
MensagemEnviado: 23 mai 2013, 02:22 
Offline

Registado: 11 mar 2013, 20:36
Mensagens: 8
Localização: Rio de Janeiro
Agradeceu: 0 vez(es)
Foi agradecido: 0 vez(es)
Opa, brigadão!
João P. Ferreira Escreveu:
(não sei se quer a prova pela definição formal)

Pô, caso não dê muito trabalho, aceitaria de bom grado x))


Topo
 Perfil  
 
MensagemEnviado: 23 mai 2013, 03:59 
Offline

Registado: 05 jan 2011, 12:35
Mensagens: 2235
Localização: Lisboa
Agradeceu: 683 vezes
Foi agradecido: 346 vezes
Pense apenas que a função seno não tem limite, confesso que não sei fazer uma demonstração mais formal...

Todavia, pode usar o limite inicial que tende para zero considerando que

\(\lim_{x \to p} f(x) = L\)

equivale a

\(\forall \delta > 0 \ \exists \epsilon > 0 \ : \ | x - p | < \epsilon \Longrightarrow | f(x) - L | < \delta\)

ou seja

\(\lim_{x \to 0} sen(1/x^2) = L\)

equivale a

\(\forall \delta > 0 \ \exists \epsilon > 0 \ : \ | x | < \epsilon \Longrightarrow | \frac{1}{x^2} - L | < \delta\)

ou seja, não consegue encontrar \(\epsilon > 0\) que respeite a condição

_________________
João Pimentel Ferreira
 
Partilhe dúvidas e resultados, ajude a comunidade com a sua pergunta!
Não lhe dês o peixe, ensina-o a pescar (provérbio chinês)
Fortalecemos a quem ajudamos pouco, mas prejudicamos se ajudarmos muito (pensamento budista)


Topo
 Perfil  
 
Mostrar mensagens anteriores:  Ordenar por  
Fazer Nova Pergunta Responder a este Tópico  [ 4 mensagens ] 

Os Horários são TMG [ DST ]


Quem está ligado:

Utilizadores a ver este Fórum: Nenhum utilizador registado e 45 visitantes


Criar perguntas: Proibído
Responder a perguntas: Proibído
Editar Mensagens: Proibído
Apagar Mensagens: Proibído
Enviar anexos: Proibído

Pesquisar por:
Ir para: